Difference between revisions of "2017 AIME II Problems/Problem 7"

(Solution)
Line 4: Line 4:
 
==Solution==
 
==Solution==
 
<math>\boxed{501}</math>
 
<math>\boxed{501}</math>
 +
 +
kx=(x+2)^2
 +
x^2+(4-k)x+4=0 ...(1)
 +
the equation has solution
 +
D=(4-k)^2-16=k(k-8)>=0
 +
so k<=0 or k>=8
 +
becuase k can't be zero or the original equation will be meanless.
 +
there are 3 cases
 +
 +
1:k=8
 +
then x=2, which is satisified the question.
 +
 +
2:k<0
 +
then one solution of the equation(1) should be in (-2,0) and another is out of it or the origin equation will be meanless.
 +
then we get 2 inequities
 +
-2<( k-4 + sqrt( k(k-8) ) )/2<0
 +
( k-4 - sqrt( k(k-8) ) )/2<-2
 +
notice k<0<sqrt(k(k-8)) and (4-k)^2=k(k-8)+16>k(k-8)
 +
we know in this case, there is always and only one solution for the orign equation.
 +
 +
3:k>8
 +
similar to case2 we can get inequity
 +
( k-4 - sqrt( k(k-8) ) )/2 < 0 <( k-4 + sqrt( k(k-8) ))/2
 +
and there are always 2 solution for the origin equation, so this case is not satisfied.
 +
 +
so we get k<0 or k=8
 +
 +
because k belong to [-500,500], the answer is 501
  
 
=See Also=
 
=See Also=
 
{{AIME box|year=2017|n=II|num-b=6|num-a=8}}
 
{{AIME box|year=2017|n=II|num-b=6|num-a=8}}
 
{{MAA Notice}}
 
{{MAA Notice}}

Revision as of 15:33, 23 March 2017

Problem

Find the number of integer values of $k$ in the closed interval $[-500,500]$ for which the equation $\log(kx)=2\log(x+2)$ has exactly one real solution.

Solution

$\boxed{501}$

kx=(x+2)^2 x^2+(4-k)x+4=0 ...(1) the equation has solution D=(4-k)^2-16=k(k-8)>=0 so k<=0 or k>=8 becuase k can't be zero or the original equation will be meanless. there are 3 cases

1:k=8 then x=2, which is satisified the question.

2:k<0 then one solution of the equation(1) should be in (-2,0) and another is out of it or the origin equation will be meanless. then we get 2 inequities -2<( k-4 + sqrt( k(k-8) ) )/2<0 ( k-4 - sqrt( k(k-8) ) )/2<-2 notice k<0<sqrt(k(k-8)) and (4-k)^2=k(k-8)+16>k(k-8) we know in this case, there is always and only one solution for the orign equation.

3:k>8 similar to case2 we can get inequity ( k-4 - sqrt( k(k-8) ) )/2 < 0 <( k-4 + sqrt( k(k-8) ))/2 and there are always 2 solution for the origin equation, so this case is not satisfied.

so we get k<0 or k=8

because k belong to [-500,500], the answer is 501

See Also

2017 AIME II (ProblemsAnswer KeyResources)
Preceded by
Problem 6
Followed by
Problem 8
1 2 3 4 5 6 7 8 9 10 11 12 13 14 15
All AIME Problems and Solutions

The problems on this page are copyrighted by the Mathematical Association of America's American Mathematics Competitions. AMC logo.png